LSAT and Law School Admissions Forum

Get expert LSAT preparation and law school admissions advice from PowerScore Test Preparation.

User avatar
 Dave Killoran
PowerScore Staff
  • PowerScore Staff
  • Posts: 5853
  • Joined: Mar 25, 2011
|
#88173
Complete Question Explanation
(The complete setup for this game can be found here: lsat/viewtopic.php?f=170&p=88166#p88166)

The correct answer choice is (A)

The best strategy on List questions is to use the individual rules of the game to eliminate incorrect answer choices. In this case, apply the rules in reverse order as that is the easiest way to apply them visually.

Answer choice (B) can be eliminated because when Q receives two stars, more than one CD receives more stars than Q, a violation of the fourth rule.

Answer choice (C) can be eliminated because according to the third rule I must receive the same rating as H or R.

Answer choice (D) can be eliminated because from the second rule H must receive exactly one more star than N.

Answer choice (E) can be eliminated because more than one CD receives more stars than Q, a violation of the fourth rule.

Answer choice (A) is thus proven correct by process of elimination.

Get the most out of your LSAT Prep Plus subscription.

Analyze and track your performance with our Testing and Analytics Package.